Ask your own question, for FREE!
Mathematics 87 Online
OpenStudy (anonymous):

Calculus BC problem... Can anyone please help? Image to problem included. Edit: I got to find the equation for S part, but Idk how to find t.

OpenStudy (anonymous):

OpenStudy (anonymous):

I'd appreciate if you can look over my proof so far...? Please.

OpenStudy (amistre64):

t is the independant variable variable i assume this is modeling with diffy qs?

OpenStudy (anonymous):

From the picture above, I wrote the equation for arc length \[S=\int\limits_{0}^{\theta _{f}} \sqrt{(\frac{ dx }{ d \theta })^2+(\frac{ dy }{ d \theta })^2 + (\frac{ dz }{ d \theta })^2} d \theta\]

OpenStudy (anonymous):

Yes, it is independent! :D

OpenStudy (anonymous):

Given: We also know that \[r^2 \theta= constant\] Where r^2 theta is with respect to d theta. Constant with respect to t. That was the relationship.

OpenStudy (amistre64):

im not aware of the physical formulas for this, but i do know that the angular velocity of rotation increases as a mass is pulled closer to its center of gravity. do we know a formula for this process?

OpenStudy (amistre64):

the time it takes L to wrap depends on the angular velocity is what i would think but im bad at physics

OpenStudy (anonymous):

\[S=\int\limits_{0}^{\theta _{f}} \sqrt{(\frac{ dx }{ d \theta })^2+(\frac{ dy }{ d \theta })^2 + (\frac{ dz }{ d \theta })^2} d \theta\] I took the derivative of X, Y, and Z. and plug in \[S=\int\limits_{0}^{\theta _{f}} \sqrt{(-R sin \theta)^2+(\frac{-d}{2 \pi})^2 + (Rcos\theta)^2} d \theta\]

OpenStudy (amistre64):

i dont have a clear idea in my head on how to approach a solution just yet, so these are some of my initial concerns :)

OpenStudy (anonymous):

Then, I use trig identities cos^2 + sin^2 =1 multiply by r^2 Rcos^2 + Rsin^2 = R^2 If I plug this in the equation I'd just get: R^2 + (d/2pi)^2 inside the parenthesis. Sorry I messed up on the latex coding so I didn't put it in the equation...

OpenStudy (amistre64):

latex is just eyecandy .... as long sa its readable its fine

OpenStudy (anonymous):

should be R^2cos^2 + R^2sin^2 = R^2. sorry

OpenStudy (amistre64):

is arclength your own idea, or was it a hint of some sort? at the moment i cant really see if thats a good approach or not.

OpenStudy (anonymous):

So I found S, but the problem is to fnd t... Idk how to set up the next step >.<

OpenStudy (anonymous):

Oh my teach said to assume that the ball is thrown at 90 degrees angle. Observation: -no tangential speed, only gravity pulling down

OpenStudy (amistre64):

t is determined by the physics formulas that govern the situation .... length of rope, angular velocity, mass, etc ...

OpenStudy (anonymous):

and the distance d is constant. We can also assume that L = total lenth S= arc length Then each time the tetherball wrap around the pole, the remaining lenth would be m= small length m= L-S idk if this is right?

OpenStudy (anonymous):

Oh no, I think he only wants to use Calc on this. No velocity or anything. Said the final answer should not have theta in it...

OpenStudy (anonymous):

When I integrate to get S, I found that S is equal to \[S= \theta _{F}\sqrt{R^2 + \frac{ d^2 }{ 4\pi^2 }}\] Where theta F is the final angle where r ends and touch the pole.

OpenStudy (amistre64):

my thoughts, and i do tend to overthink these things. angular momentum is conserved in a system. this is something that kepler oberved in planetary motion. the area of the sweep remains constant as the planets move in their orbits. this means that the planets actually speed up and slow down as they travel in their orbits in order to maintain keplers law. the ball around the pole would have to obey the same principle, which is why if youve ever played tetherball the angular velocity speeds up as the ball gets closer to the pole. also why an iceskater can do those amazing spins ... they pull in thier arms and their angular momentum increases in order to maintain this conservation of mometum.

OpenStudy (amistre64):

needless to say im not going to be much use in finding a suitable solution .... modeling with diffy qs is not a strong point.

OpenStudy (anonymous):

I see.. thank you for your help! >.< Ik that physics would be so much easier to solve this. Idk why but my teacher wants it in calc with xyz haha.

OpenStudy (amistre64):

good luck :)

OpenStudy (anonymous):

Thank you :D You're amazing for helping me out!

OpenStudy (superdavesuper):

The key is what ur teach told u: "no tangential speed, only gravity pulling down" I will change the naming quite a bit so please look closely at the pic below as the variables may not be defined the same as urs: |dw:1427653488457:dw| l is the length of string ALREADY wrapped on the cylinder at time t so l=0 at t=0; when l=L, t=the answer n lets call it T. if the string is unwrapped from the cylinder at time t, we get the triangle above. as it only gravity pulling down, from physics we know y=1/2*g*t^2 where g is gravity. from the angular velocity w, x=2*pi*R*w*t it is a right-angled triangle, so x^2 + y^2 = l^2 So at time T, l=L, x=2*pi*R*w*T, y=1/2*g*T^2 (2*pi*R*w*T)^2 + (1/2*g*T^2)^2 = L That would the equation between T and L.

Can't find your answer? Make a FREE account and ask your own questions, OR help others and earn volunteer hours!

Join our real-time social learning platform and learn together with your friends!
Latest Questions
LaPrincessaHermosa: I decided to edit Jazmine and Tiana. What y'all think?
42 seconds ago 11 Replies 2 Medals
Can't find your answer? Make a FREE account and ask your own questions, OR help others and earn volunteer hours!

Join our real-time social learning platform and learn together with your friends!